3
$\begingroup$

I am considering a combinatorial argument which involves the following quantity. We use the prime counting function $\pi(n)$ and to save on exponents we set $h=\pi(n/2)$. The quantity as a function of integer $n \gt 7$ is $$(\pi(n)!)^{1/(n-h)}$$

Computations for small $n$ suggest this is always less than $4$, as do rough back-of-the-envelope asymptotic calculations. Is this bounded above for all $n \gt 7$? If so, what is the bound? (I'm hoping it is always less than 3.)

Gerhard "Researching Minds Want To Know" Paseman, 2020.05.30.

$\endgroup$
6
  • $\begingroup$ nice question ,upvote $\endgroup$ May 31, 2020 at 2:06
  • $\begingroup$ I think it is upper bounded by 4 using a rough approximation to the prime counting function. I would appreciate verification. Gerhard "Worth An Acknowledgement To Me" Paseman, 2020.05.30. $\endgroup$ May 31, 2020 at 2:16
  • 2
    $\begingroup$ Don't you just want to plug an effective form of the prime number theorem into Stirling's formula here? $\endgroup$
    – Will Sawin
    May 31, 2020 at 2:31
  • 4
    $\begingroup$ Your rough back-of-the-envelope asymptotic calculations suggest the limit is $e$, right? I think these calculations, unless you did them in a very strange way, are already a proof. $\endgroup$
    – Will Sawin
    May 31, 2020 at 2:35
  • $\begingroup$ I often do strange things @Will. I guess I am just looking for confirmation. Gerhard "Anything For These Uncertain Times" Paseman, 2020.05.30. $\endgroup$ May 31, 2020 at 2:38

3 Answers 3

8
$\begingroup$

Let $k:=\pi(n)$, so that $p_k\le n<p_{k+1}$, where $p_k$ is the $k$th prime. By the last displayed formula in this section of the Wikipedia article, \begin{equation*} -1+\ln(k\ln k)<\frac{p_k}k<\ln(k\ln k) \end{equation*} if $k\ge6$, whence \begin{equation*} n>-k+k\ln(k\ln k),\quad n/2<m_k:=\frac{k+1}2\,\ln((k+1)\ln(k+1)). \end{equation*} Therefore, letting \begin{equation*} c_1:=1.25506,\quad r(k):=\frac{\ln((k+1)\ln(k+1))}{\ln m_k} \end{equation*} and using this result, we get \begin{equation*} h=\pi(n/2)\le\pi(m_k)<c_1\frac{m_k}{\ln m_k}=c_1\frac{k+1}2\,r(k). \end{equation*} Next, \begin{multline*} r(k):=\frac{\ln(k+1)+\ln\ln(k+1)}{\ln(k+1)+\ln[\ln(k+1)+\ln\ln(k+1)]-\ln2} \\ <\frac{\ln(k+1)+\ln\ln(k+1)}{\ln(k+1)+\ln\ln(k+1)-\ln2}<\frac{10}9 \end{multline*} if \begin{equation*} k\ge195, \end{equation*} which will be assumed henceforth.
So,\begin{equation*} h<c_2(k+1), \end{equation*} where \begin{equation*} c_2:=\frac7{10}>c_1\frac{10}9\Big/2. \end{equation*} So, using the trivial inequality $k!\le k^k$, we have \begin{equation*} \ln[(\pi(n)!)^{1/(n-h)}]=\frac{\ln(k!)}{n-h} \le\frac{k\ln k}{-k+k\ln(k\ln k)-c_2(k+1)} <\frac{k\ln k}{k\ln k}=1 \end{equation*} and hence \begin{equation*} (\pi(n)!)^{1/(n-h)}<e \tag{1} \end{equation*} for $k\ge239$, that is, for $n\ge1499$. By direct calculation, (1) holds for $n\le1498$ as well. It is also easy to see that the upper bound $e$ on $(\pi(n)!)^{1/(n-h)}$ is exact.

$\endgroup$
5
  • $\begingroup$ Thanks for this! I will go over this and cite it in my write-up. It turns out I have some leeway: h can be a little larger than pi(n/2), and it is enough if I can show a weaker upper bound of three minus epsilon or even four minus epsilon. Do you think we can simplify this given the leeway, but also arrange so that it holds for k greater than 50, so that less computation for small k is needed? (I'm willing to accept verifying k less than 40, so that c1 can drop a bit.) Gerhard "Looking Forward To The Math" Paseman, 2020.05.31. $\endgroup$ May 31, 2020 at 16:10
  • $\begingroup$ @GerhardPaseman : If you make $h$ larger, then your expression will get larger. However, if you don't need the exact upper bound, then the increase of $h$ could be OK within certain limits, and even the proof could possibly be simplified. I suspect the proof can be simplified a bit anyway, as many of the first proofs. $\endgroup$ May 31, 2020 at 16:20
  • $\begingroup$ I have indeed simplified the proof a bit. $\endgroup$ May 31, 2020 at 17:02
  • $\begingroup$ Thank you again. I will upvote and accept this later. It turns out I have a simpler argument and simpler quantity to estimate which achieves my goal. I will post later after I have checked it . Gerhard "Can it Be So Simple?" Paseman, 2020.05.31. $\endgroup$ May 31, 2020 at 19:16
  • $\begingroup$ I have further simplified the answer, which now does not use any version of Stirling's formula. $\endgroup$ May 31, 2020 at 19:31
1
$\begingroup$

So I accept the answer of Iosif Pinelis, but it turns out I don't need the exact result now. I will post some backstory, and then the reason I don't need it now.

Thanks to MathOverflow user Daniel.W, and his question (360323) on strengthening Sylvester's theorem, I've been motivated to read the paper On Arithmetical Series. I approached it after a hint from Emil Jerabek to look at the thesis of Alan Woods. The thesis contained a write-up of Sylvester's method that I finally understood, and this allowed me to try to understand some of the proof in the 1892 paper.

A different version appears in a 1929 paper of Schur (which I have yet to find) and a (mostly) combinatorial one in a paper of Erdos in 1934. However, the arguments are still involved, and the Erdos paper leaves a lot of finitely many exceptions to be explored to yield a full proof.

After looking at the basic relation in Sylvester's paper, I (re-)discovered a result that allowed one to show that there was a number in (m,m+,n] with a prime factor greater than n whenever 4m was at least as big as n^2. This was heartening since previously I could only show it for m bigger than exponential in n. This in turn allowed me to discover a method which involved the quantity in the question above, and simple arguments showed that m only needed to be bigger than a small constant times n. (Iosif's argument and some additional computation shows the small constant is 3.). I then was going to attempt a third method to bridge the remaining gap which is for all m at least n. All of this was then going to be retooled to answer the motivating MathOverflow question.

After seeing Iosif's argument and thinking on simplifying the motivating argument, I found it. Here it is.

Write the product of the integers in (m,m+n] as P=(m+1)...(m+n). Rewrite as W(n!)L, where W are the prime factors of P/(n!) which are at most n gathered together, and L is the product of all the prime factors larger than n.

A key observation of Sylvester, (which I invite the reader to prove) is that W is at most (and for n bigger than 7, strictly less than) (m+n-p+1)...(m+n), where p is $\pi(n)$. This is because W is the product of p distinct prime powers, each one dividing a term of P (and usually different powers divide different terms, we suffer no loss in assuming this).

So if (m,m+n] has only n-smooth numbers, then L=1 and n! Is bigger (not necessarily strictly) than (m+1)...(m+n-p). The literature now expends a lot of effort to show how small m is, and Sylvester himself resorts to existence of primes in (m, 3m/2] to complete his argument. There is an easier way, however.

Write m=jn+i for i non negative. Then rewrite W(n!) = P by dividing out W and dividing out terms in the factorial larger than p. We get p! greater than j^(n-p), if P is n-smooth.

But we can argue with Chebyshev estimates to get j less than 6, and if we are as thorough as Iosif we can get j less than 3 with a small amount of computation needed. I need to perform this step but I believe p less than 50 should be more than sufficient.

So when the dust settles, we have reduced a large portion of Sylvester's argument to showing log(p!) Is less than n-p, using nothing more than grade school arithmetic and Sylvester's observation on W. With care we get that (m,m+n] has a multiple of a prime greater than n when m is at least 3n or greater. If need be, we can turn to the Erdos proof to handle m smaller than 3n.

However, there is more. The motivating question asks for two distinct numbers in the interval which have prime factors bigger than n. We now let L be a product of d many candidates for fixed d of members of (m,m+n]. We now are comparing log(p!) to n-p-d, and we will get the same bound on j, although this bound may start holding for larger m only.

Given the amount of time I spent reading these proofs, I'm surprised not to find this observation (that j is less than 3) is in the literature. We can use this observation, Chebyshev estimates, and work of Nagura or earlier to answer the motivating question affirmatively. That C=18 for two numbers hasn't been proved yet.

Gerhard "Is Confident It Will Be" Paseman, 2020.05.31.

$\endgroup$
1
  • $\begingroup$ Now, seeing Iosif's replacement, of p^p. the argument has become simpler. Gerhard "Many Thanks Again, Iosif Pinelis" Paseman, 2020.05.31. $\endgroup$ May 31, 2020 at 21:26
0
$\begingroup$

It seems my thinking gets organized after posting the question.

The natural logarithm of the quantity $\pi(n)!$ is near $\pi(n)\log(\pi(n)/e) + (\log(\tau\pi(n)))/2$ (a number-theoretic use for $\tau$, the circumference of a unit radius circle). Using an approximation to $\pi(n)$ we get that this is less than $An$ for some $A \lt 2$. But $An/(n-h)$ is bounded above by $2A$, and gets very close to $A$. So with some work the original quantity should be shown to be less than $e^A$.

Verification is still appreciated.

Gerhard "And Still Worth An Acknowledgement" Paseman, 2020.05.30.

$\endgroup$
3
  • $\begingroup$ It may help to set $k= \pi(n)$ and use upper bounds for the $n$th prime to deduce $n> k \log k+ k \log \log k - k$ because then the numerator and denominator of the logarithm will look quite similar although $\pi(n/2)$ then becomes annoying $\endgroup$
    – Will Sawin
    May 31, 2020 at 2:47
  • $\begingroup$ I may do that. however, the important thing for me is that $h$ is small relative to $\pi(n)$. Gerhard "Going To Write It Up" Paseman, 2020.05.30. $\endgroup$ May 31, 2020 at 2:50
  • $\begingroup$ For the sake of history, I compare $\pi(n)!$ to $k^{n-h}$. If $k=3$ makes that quantity too large, then the conclusion of Sylvester's Theorem follows for his $m \gt 2n$. This also helps toward a generalization I am working on with another MathOverflow user. Gerhard "Not Doing It For Power" Paseman, 2020.05.30. $\endgroup$ May 31, 2020 at 3:14

Your Answer

By clicking “Post Your Answer”, you agree to our terms of service and acknowledge you have read our privacy policy.

Not the answer you're looking for? Browse other questions tagged or ask your own question.